Pulmonary Disorders Flashcards

1
Q

Questions 1 and 2 pertain to the following case.
A 20-year-old woman presents to the clinic with an
asthma exacerbation. She states that she has been using
her boyfriend’s albuterol inhaler on a regular basis for
the past 2 years. During the past few months, she has
been using the inhaler on a daily basis and sometimes
at night.
1. Which description best classifies her asthma
severity?
A. Mild intermittent.
B. Mild persistent.
C. Moderate persistent.
D. Severe persistent.

A
  1. Answer: C
    Answer A is incorrect because patients with mild inter-
    mittent have symptoms 2 days or fewer per week. This
    patient uses the inhaler throughout the day every day
    and sometimes at night, indicating that she has daily
    symptoms. Answer B is incorrect because the patient
    is having daily symptoms, and patients with mild per-
    sistent asthma have symptoms more often than 2 days
    per week but fewer than daily. Answer C is correct
    because the patient is having daily symptoms. Answer
    D is incorrect because “severe persistent” means that
    the frequency of symptoms is throughout the day,
    nighttime symptoms often occur seven times a week,
    a SABA is needed several times a day, normal activity
    is extremely limited, the FEV1
    is less than 60% of pre-
    dicted, and the FEV1
    /FVC is reduced by more than 5%.
How well did you know this?
1
Not at all
2
3
4
5
Perfectly
2
Q
  1. Which is the best asthma maintenance therapy for
    this patient?
    A. Fluticasone low dose.
    B. Montelukast.
    C. Fluticasone low dose plus salmeterol.
    D. Fluticasone high dose.
A
  1. Answer: C
    Moderate asthma requires step 3 treatment for control.
    Step 3 preferred treatment is an inhaled steroid (low
    dose) plus a LABA. Answer A is incorrect because low-
    dose fluticasone is considered step 2 treatment. Answer
    B is incorrect because montelukast is an alternative
    treatment for step 2. Answer C is correct; fluticasone
    low dose plus salmeterol is the preferred step 3 treat-
    ment. Answer D is incorrect because high-dose flutica-
    sone monotherapy is not recommended for treatment of
    any step of asthma treatment.
How well did you know this?
1
Not at all
2
3
4
5
Perfectly
3
Q
  1. Which type of data best classifies the number of
    times a low-dose inhaled corticosteroid/formoterol
    inhaler is used in 1 month?
    A. Nominal.
    B. Ordinal.
    C. Interval.
    D. Ratio.
A
  1. Answer: D
    Answer A is incorrect because nominal data contain no
    quantitative value and are simply labels. Ordinal data is
    a measure of non-numeric concepts in which the order
    of the value is important but the differences between
    values are not truly known, making Answer B incorrect.
    Answer C is incorrect because although interval data
    are ranked in a specific order with a consistent level of
    magnitude difference between units, there is no absolute
    zero. Answer D is correct because ratio data are ranked
    in a specific order with a consistent level of magnitude
    difference between units, with an absolute zero.
How well did you know this?
1
Not at all
2
3
4
5
Perfectly
4
Q
  1. You are designing a study in which you will com-
    pare the percentages of patients with an asthma-
    related hospitalization receiving fluticasone/
    salmeterol and those receiving fluticasone alone.
    Which statistical test is best for analyzing this
    comparison?
    A. Analysis of variance (ANOVA).
    B. Chi-square.
    C. Mann-Whitney U test.
    D. Unpaired t-test.
A
  1. Answer: B
    The percentage of patients receiving fluticasone/salme-
    terol who have an asthma-related hospitalization will
    be compared with the percentage of patients receiving
    fluticasone who have an asthma-related hospitaliza-
    tion. We assume that these two groups are normally
    distributed. These data are considered nominal. Use
    of ANOVA is appropriate when there are more than
    two treatment groups; therefore, Answer A is incor-
    rect. Answer B is correct because the chi-square test
    is appropriate for analyzing nominal or categorical
    data. The Mann-Whitney U test is appropriate when
    continuous data are not normally distributed; therefore,
    Answer C is incorrect. Answer D is incorrect because
    the Student unpaired t-test is used for continuous data
    that are normally distributed.
How well did you know this?
1
Not at all
2
3
4
5
Perfectly
5
Q
  1. A 22-year-old woman with asthma is taking an
    albuterol metered-dose inhaler (MDI) 2 puffs as
    needed and fluticasone 110 mcg/puff MDI 2 puffs
    twice daily. She received the influenza vaccine
    during last year’s influenza season, completed the
    mRNA coronavirus disease 2019 (COVID-19) vac-
    cine series, and her last tetanus vaccine (tetanus,
    diphtheria, and pertussis [Tdap]) was at age 17;
    there is no documentation of her having received a
    pneumococcal vaccine. Which is the best vaccine
    for her to receive at her next family medicine clinic
    appointment, scheduled for July?
    A. Influenza.
    B. Pneumococcal.
    C. Tetanus and diphtheria (Td).
    D. Herpes zoster.
A
  1. Answer: B
    Answer A is incorrect because the influenza vaccine
    is recommended in people with chronic cardiovascular
    or pulmonary diseases such as asthma. However, usu-
    ally the influenza vaccine is given in the fall or early
    winter to offer protection when the risk of infection is
    highest. Answer B is correct; the pneumococcal vac-
    cine is recommended in people 19–64 years of age with
    asthma, and this patient falls into this category. Answer
    C is incorrect because the tetanus booster (Td) is rec-
    ommended every 10 years, and it has not been 10 years
    since this patient’s last Td, which was given as Tdap.
    The herpes zoster vaccine is recommended by the CDC
    for adults 50 and older so Answer D is incorrect.
How well did you know this?
1
Not at all
2
3
4
5
Perfectly
6
Q
  1. A 60-year-old man is diagnosed with chronic
    obstructive pulmonary disease (COPD) today. He
    reports experiencing symptoms most days of the
    week. His Modified Medical Research Council
    (mMRC) score is 1. His spirometry reveals a forced
    expiratory volume in 1 second (FEV1
    ) of 70% of
    predicted and an FEV1
    /forced vital capacity (FEV1
    /
    FVC) of 60% of predicted. He has had no previous
    COPD exacerbations. Which medication is best to
    initiate?
    A. Inhaled fluticasone.
    B. Inhaled tiotropium.
    C. Inhaled fluticasone/salmeterol.
    D. Oral roflumilast.
A
  1. Answer: B
    This patient is in group A, based on having no exacerba-
    tions and an mMRC score of 1. A single LA bronchodi-
    lator is the best choice of medication treatment. Answer
    A is incorrect because an ICS is considered only in
    patient group E with elevated eosinophils and should
    never be used as monotherapy in COPD. Tiotropium
    is a LA bronchodilator (anticholinergic) that would be
    appropriate in this patient, making Answer B correct.
    (A LABA would also be appropriate, but it was not one
    of the choices.) Answer C is incorrect because the ICS/
    LABA combination is generally not preferred for treat-
    ment of COPD. Answer D is incorrect because roflumi-
    last is only indicated in severe COPD (FEV1 less than
    50% of predicted) associated with chronic bronchitis
    when the patient has a history of frequent exacerbations
How well did you know this?
1
Not at all
2
3
4
5
Perfectly
7
Q
  1. A 75-year-old woman with a 50 pack-year history of
    tobacco use presents to her primary care physician
    with a complaint of worsening shortness of breath
    over the past 6 months. She has no documented
    history of asthma or COPD. Which symptom is
    most consistent with the diagnosis of COPD?
    A. Dyspnea that is progressive, persistent, and
    worse with exertion.
    B. FEV1
    /FVC greater than 70% of predicted.
    C. Chronic cough present only at night.
    D. COPD Assessment Test (CAT) score of 5.
A
  1. Answer: A
    Answer A is correct because dyspnea that is progres-
    sive, persistent, and worse with exertion is a hallmark
    symptom of COPD. Answer B is incorrect because spi-
    rometry findings of an FEV1
    /FVC less than 70% of pre-
    dicted are required for a diagnosis of COPD. Answer C
    is incorrect because although a chronic cough is usually
    a symptom of COPD, this cough is rarely only nocturnal
    and is most often present throughout the day. The CAT
    has a score range of 5–30, with higher scores indicat-
    ing worse COPD. A CAT score of 5 is within the upper
    limit of normal scores, making Answer D incorrect.
How well did you know this?
1
Not at all
2
3
4
5
Perfectly
8
Q
  1. A 68-year-old man was given a diagnosis of COPD
    1 year ago, and his symptoms have been managed
    with an albuterol inhaler as needed and umeclidin-
    ium/vilanterol since. He reports adherence to cur-
    rent regimen with appropriate inhaler technique.
    Recent spirometry revealed an FEV1
    /FVC less
    than 70% and FEV1
    of 65%. Last week, he was dis-
    charged from the hospital after his second COPD
    exacerbation in the past year. His current mMRC
    score is 1 and blood eosinophil count is 212 cells/
    mcL. Which maintenance therapy is most appro-
    priate for this patient?
    A. Continue umeclidinium/vilanterol.
    B. Continue umeclidinium/vilanterol and add
    roflumilast.
    C. Continue umeclidinium/vilanterol and add
    fluticasone.
    D. Discontinue umeclidinium/vilanterol and start
    fluticasone furoate/umeclidinium/vilanterol.
A
  1. Answer: D
    This patient has had two exacerbations in the past year
    on dual bronchodilator therapy and therefore should
    follow the exacerbation pathway. The patient is cur-
    rently taking a LABA/LAMA combination and should
    be escalated to LABA/LAMA/ICS based on a blood
    eosinophil count > 100 cells/mcL, making Answer D
    correct. Answer A is incorrect because this option does
    not escalate therapy. Answer B is incorrect because
    roflumilast is considered if a blood eosinophil count is
    < 100 cells/mcL, FEV1
    is < 50%, and chronic bronchi-
    tis is present. Answer C is incorrect because although
    adding an ICS is appropriate, a single inhaler is pre-
    ferred because it may be more convenient and effective.
    In addition, a single ICS inhaler can potentially lead to
    ICS monotherapy, which is not appropriate in patients
    with COPD.
How well did you know this?
1
Not at all
2
3
4
5
Perfectly
9
Q
  1. A 32-year-old woman, 30 weeks pregnant, is seen
    for a routine prenatal check today (October). This
    is her second pregnancy in 3 years. During her
    first pregnancy, she received Tdap at 36 weeks’
    gestation; her last tetanus booster was at age 21.
    She received all of her routine childhood immuni-
    zations and is up to date with COVID-19 vaccines.
    Which immunizations would be best for her today?
    A. Td.
    B. Td and influenza.
    C. Tdap.
    D. Tdap and influenza.
A
  1. Answer: D
    During each pregnancy, it is recommended that the
    woman receive a Tdap vaccination, regardless of the
    time since her last Td or Tdap immunization. Therefore,
    Answers A and B are incorrect because they do not
    include Tdap. Answer C is incorrect because an annual
    influenza vaccine is recommended for all pregnant
    women, and the visit is taking place in October, when
    it would be appropriate to administer the influenza vac-
    cine. Answer D is correct because it includes both Tdap
    and influenza.
How well did you know this?
1
Not at all
2
3
4
5
Perfectly
10
Q
  1. A 21-year-old man is seen for his annual wellness
    exam in July. He received an influenza vaccine
    last October but states that it was his first immu-
    nization in about 10 years, besides completing the
    mRNA COVID-19 vaccine series. He works full-
    time at an office job that requires no travel. Which
    vaccines would be best for him today?
    A. Tdap, human papillomavirus (HPV).
    B. Tdap, influenza, meningococcal polysaccharide.
    C. Td, meningococcal polysaccharide, HPV.
    D. Td, influenza.
A
  1. Answer: A
    It is recommended that a one-time dose of Tdap be
    substituted for Td in all patients and that the HPV vac-
    cine be administered to males up to age 45. Therefore,
    Answer A is correct. Answer B is incorrect because
    it is inappropriate to administer the influenza vaccine
    in July, and the patient has no risk factors indicating
    that he would benefit from the meningococcal vaccine
    (e.g., he is not living in a college dormitory and does not
    travel frequently). Answer C is incorrect because the
    patient should receive Tdap in place of the Td booster
    and does not need the meningococcal vaccine. Answer
    D is incorrect because the patient should receive Tdap
    in place of the Td booster, and it is not appropriate to
    administer the influenza vaccine in July
How well did you know this?
1
Not at all
2
3
4
5
Perfectly
11
Q

Patient Cases
Questions 1–3 pertain to the following case.
A 23-year-old woman has been coughing and wheezing about twice weekly, and she wakes up at night about three
times per month. She has never been given a diagnosis of asthma and has not been to a physician, she says, “in
years.” She uses her roommate’s albuterol inhaler, but having recently run out of refills, she is seeking care. Her
activities are not limited by her symptoms. Spirometry today reveals FEV1
82% of predicted.
1. According to the NAEPP guidelines, which is the best classification of her asthma?
A. Intermittent.
B. Mild persistent.
C. Moderate persistent.
D. Severe persistent.

A
  1. Answer: B
    Her symptom frequency of twice weekly, her FEV1
    of
    more than 80% of predicted (normal), and the lack of
    interference with activity are consistent with intermit-
    tent asthma. Answer A is incorrect; her night awak-
    enings for asthma symptoms occur three times per
    month, which is consistent with mild persistent asthma.
    Therefore, Answer B is correct. The specific level of
    persistent asthma is based on the most severe category
    met. Answers C and D are incorrect because only one
    of her signs and symptoms falls under mild persistent
    asthma (nighttime awakenings), and the rest fall under
    intermittent asthma.
How well did you know this?
1
Not at all
2
3
4
5
Perfectly
12
Q
  1. According to GINA guidelines, which medication is best to recommend for her, in addition to budesonide/
    formoterol MDI every 4–6 hours as needed?
    A. No additional therapy needed.
    B. Oral montelukast 10 mg/day.
    C. Mometasone DPI 220 mcg/puff 2 puffs daily.
    D. Budesonide/formoterol MDI 80/4.5 mcg per puff 2 puffs twice daily.
A
  1. Answer: A
    Because she has mild persistent asthma, step 2 is rec-
    ommended for initial treatment. Answer A is correct
    given that step 2 treatment involves adding an as-needed
    low-dose ICS-formoterol. Answer B is incorrect
    because montelukast is an alternative therapy (not first
    line) for step 2. Answer C is incorrect; mometasone 220
    mcg 2 puffs once daily is a medium-dose ICS (alterna-
    tive step 3). Budesonide/formoterol at the dose listed is
    a low-dose ICS plus a LABA (step 3 therapy), making
    Answer D incorrect.
How well did you know this?
1
Not at all
2
3
4
5
Perfectly
13
Q
  1. At first, the patient’s symptoms were well controlled on your recommended therapy. However, when winter
    arrived, her symptoms were no longer well controlled, and she started using her budesonide/formoterol MDI
    3 or 4 days per week during the day. Which is the preferred treatment change?
    A. No change in therapy is needed.
    B. Add budesonide/formoterol MDI 80/4.5 mcg per puff 2 puffs twice daily.
    C. Add montelukast orally 10 mg daily.
    D. Add mometasone DPI to 220 mcg/puff 2 puffs daily.
A
  1. Answer: B
    Her asthma is classified as not well controlled because
    of the frequency of her daytime symptoms and her need
    to use albuterol more than 2 days/week. When asthma
    is classified as not well controlled, the recommendation
    is to increase to the next step of treatment. The patient
    would now be classified as needing step 3 treatment.
    The recommended treatment for step 3 is a low-dose
    ICS-formoterol. Answer A is incorrect because the rec-
    ommended treatment for step 3 is a daily low-dose ICS
    plus a LABA. Answer B is correct because it results in
    single maintenance and reliever therapy (SMART) with
    ICS-formoterol reliever (step 3 therapy). Adding mon-
    telukast is an alternative therapy for step 2 and would
    not be the preferred option, making Answer C incor-
    rect. Answer D is incorrect; mometasone 220 mcg/puff
    2 puffs daily is a medium-dose ICS.
How well did you know this?
1
Not at all
2
3
4
5
Perfectly
14
Q
  1. A 25-year-old man presents to the ED with shortness of breath at rest. He is having trouble with conversation.
    He used 4 puffs of albuterol MDI at home, with no resolution of symptoms. His FEV1
    is 38% of predicted.
    Which therapy, in addition to oxygen, is best for him initially in the ED?
    A. Oxygen alone is sufficient.
    B. Give albuterol MDI 8 puffs every 20 minutes for 1 hour.
    C. Give albuterol plus ipratropium by nebulizer every 20 minutes for 1 hour, plus oral corticosteroids.
    D. Give albuterol plus ipratropium by nebulizer every 20 minutes for 1 hour, plus intravenous corticosteroids.
A
  1. Answer: C
    Because the patient is experiencing shortness of breath at
    rest, has trouble with conversation, and has an FEV1
    less
    than 50% of expected, his asthma exacerbation is clas-
    sified as severe. For severe asthma exacerbations in the
    ED setting, pharmacologic treatment is recommended
    in addition to oxygen to achieve an Sao2
    of 93%–98%;
    therefore, Answer A is incorrect. Answer B is incorrect
    because a high-dose inhaled SABA should be combined
    with ipratropium and administered by either a nebulizer
    or an MDI with valved holding chamber every 20 min-
    utes for 1 hour or continuously for severe asthma exac-
    erbations in the ED setting. Answer C is correct because
    it correctly combines a SABA with ipratropium and
    includes OCSs. Answer D correctly provides a SABA
    with ipratropium; however, intravenous corticosteroids
    are not recommended unless the patient has an incom-
    plete response to inhaled and oral treatments
How well did you know this?
1
Not at all
2
3
4
5
Perfectly
15
Q

Patient Cases
5. A 62-year-old man was recently given a diagnosis of COPD. Spirometry reveals an FEV1
/FVC 60% of pre-
dicted, pre-bronchodilator FEV1
70% of predicted, and post-bronchodilator FEV1
72% of predicted. His symp-
toms are quite bothersome. He has an mMRC score of 2. He had 1 exacerbation in the past year that did not
require hospitalization. In addition to albuterol 2 puffs every 4–6 hours as needed, which pharmacotherapy
option is most appropriate to initiate?
A. Tiotropium 2.5 mg 2 puffs once daily.
B. Salmeterol 50 mcg 1 puff twice daily.
C. Tiotropium/olodaterol 2.5/2.5 2 puffs once daily.
D. Fluticasone furoate/umeclidinium/vilanterol 100 mcg/62.5 mcg/25 mcg 1 inhalation once daily.

A
  1. Answer: C
    Answer A is incorrect; according to the GOLD guide-
    lines, the recommended treatment for patient group B
    is the combination of a LABA and LAMA in addition
    to a SA bronchodilator as needed. Answer B incorrect
    because it does not include a LAMA in combination
    with the salmeterol. Answer C correct because the
    LABA/LAMA combination is recommended for initial
    treatment of group B. Answer D is incorrect because
    ICSs are only considered as part of triple therapy
    (LABA + LAMA + ICS) in group E who have a blood
    eosinophil count ≥ 300 cells/mcL.
How well did you know this?
1
Not at all
2
3
4
5
Perfectly
16
Q
  1. A 52-year-old woman with COPD reports a gradual worsening of shortness of breath during the past few years.
    Spirometry reveals FEV1
    /FVC 55% and FEV1
    63% of predicted. Her CAT score is 10 and eosinophil count is
    68 cells/mcL. She has not had a COPD exacerbation or received systemic corticosteroids in the past 2 years.
    Her current COPD medications are tiotropium inhaler once daily and albuterol HFA as needed. According to
    the GOLD guidelines, which is the most appropriate course of action?
    A. Adding salmeterol 50 mcg 1 puff twice daily.
    B. Adding long-term azithromycin 250 mg once daily.
    C. Adding fluticasone 110 mcg 2 puffs twice daily.
    D. Discontinuing tiotropium and initiating salmeterol/fluticasone 50 mcg/250 mcg 1 puff twice daily
A
  1. Answer: A
    This patient is currently being treated with a LAMA
    (tiotropium). Because her control is worsening with no
    history of recent exacerbations, she should be placed in
    the dyspnea pathway of the follow-up treatment algo-
    rithm. Because the patient is on bronchodilator mono-
    therapy, her treatment should be intensified, for which
    combined LA bronchodilators are recommended (LABA
    plus LAMA). Salmeterol is a LABA; therefore, Answer
    A is correct. Answer B is incorrect because the long-
    term use of antibiotics is recommended only in patients
    on dual bronchodilator therapy who are experiencing
    exacerbations and are former smokers with an eosin-
    ophil count < 100 cells/mcL or patients experiencing exacerbations on triple therapy who are former smokers.
    Answers C and D both contain the option of adding flu-
    ticasone in some form. Inhaled corticosteroids are rec-
    ommended only in patients experiencing exacerbations;
    therefore, Answers C and D are incorrect.
17
Q
  1. A 64-year-old woman with COPD in GOLD patient group A presents for a clinic visit. In the past few days,
    she has had a worsening of shortness of breath and a productive cough with more “cloudy” and more copious
    sputum than usual. Pulse oximetry is 95% on room air. She has a nebulizer at home. In addition to regular use
    of albuterol plus ipratropium by nebulizer every 1 to 4 hours, which is the best course of action?
    A. No additional therapy is necessary.
    B. Add oral prednisone 40 mg once daily for 5 days.
    C. Add azithromycin 500 mg daily for 3 days.
    D. Add oral prednisone 40 mg once daily for 5 days and azithromycin 500 mg daily for 3 days.
A
  1. Answer: D
    Answer A is incorrect because the patient’s symptoms
    indicate that she has a COPD exacerbation and addi-
    tional treatment is warranted. According to the latest
    GOLD guidelines, OCSs are indicated in most exac-
    erbations. The recommended dose is oral prednisone
    40 mg daily for 5 days. Antibiotic treatment is also
    indicated because the patient has all three cardinal
    symptoms of airway infection: increased sputum puru-
    lence, increased sputum volume, and increased dyspnea.
    Azithromycin is one of the recommended antibiotics.
    Although Answers B and C contain components of the
    correct treatment for this patient’s COPD exacerbation,
    neither Answer B nor Answer C is complete. Answer D
    is correct because it incorporates both an OCS and an
    antibiotic for this patient’s COPD exacerbation.
18
Q

Patient Cases
8. A 71-year-old woman with COPD is taking tiotropium (Spiriva) inhaled 1 capsule/day. She received the influ-
enza vaccine last October, her last Td vaccine was at age 65, and her PPSV23 was given at age 60. She has not
yet received the zoster vaccine, but she had an episode of severe zoster infection 5 years ago. Which is the most
appropriate choice of vaccines for the patient’s October internal medicine clinic appointment?
A. Only the influenza vaccine.
B. Influenza and PCV15 vaccines.
C. Influenza, PCV15, and RZV.
D. Influenza, PCV15, RZV, and Tdap.

A
  1. Answer: D
    The CDC recommends that the influenza vaccine be
    given every year to every person 6 months and older.
    All adults 65 and older without a history of PCV
    should receive PCV20 or PCV15. Adults with previous
    PPSV23 only may receive PCV20 or PCV15 at least 1
    year after their last PPSV23 dose. Recombinant zoster
    vaccination is recommended in all adults 50 and older,
    regardless of previous zoster infection. The Tdap vac-
    cine is recommended as a one-time replacement for Td
    in all people, now including those 65 and older. Answer
    A is incorrect because it does not address the need for
    PCV15, zoster vaccine, or Tdap. Answer B is incorrect
    because it does not address zoster or Tdap vaccination.
    Answer C is incorrect because it does not include Tdap.
    Answer D is correct because it includes all four of the
    appropriate vaccinations for October’s visit.
19
Q
  1. A 20-year-old woman who is going away to college presents for a physical examination in July. She will be
    living in the dormitory. She smokes 1/2 pack/day but has no other medical conditions. She is up to date on all
    of her routine childhood vaccines until the age of 9 but has not received any vaccines in the past 11 years. She
    is not sexually active. Which is the most appropriate choice of vaccines to be given today?
    A. Td and HPV.
    B. Tdap, MenACWY, and HPV.
    C. MenACWY, PCV20, and Td.
    D. MenACWY, PCV20, Tdap, and HPV.
A
  1. Answer: D
    First-year college students up to age 21 who live in
    dormitories, if not previously vaccinated on or after
    age 16, should receive the meningococcal vaccine
    (MenACWY; Menactra, Menveo). The PCV20 is rec-
    ommended for smokers 19–64 years of age, and this
    patient is a smoker. The Td vaccine should be given
    every 10 years. In adults younger than 65, a one-time
    dose of Tdap should be given, regardless of the time
    interval since the most recent tetanus vaccination. The
    HPV vaccine is recommended for all people 9–26 years
    of age. Ideally, it should be given before the start of sex-
    ual activity, but it should still be administered to sexu-
    ally active individuals. Answer A is incorrect because
    it does not include a meningococcal or pneumococcal
    vaccine and recommends Td instead of Tdap. Answer
    B is incorrect because it does not include the pneumo-
    coccal vaccine. Answer C is incorrect because it does
    not include HPV vaccine and recommends Td instead
    of Tdap. Answer D appropriately includes all four nec-
    essary immunizations for the patient’s visit today and
    is correct
20
Q
  1. A 21-year-old man with type 1 diabetes presents for an influenza vaccine. He has an egg allergy, with
    angioedema and difficulty breathing after eating eggs. Which is the most appropriate influenza vaccination
    plan for this patient, assuming all influenza vaccines are available?
    A. IIV, RIV, or live-attenuated influenza vaccine may be used in any setting.
    B. Either IIV or RIV should be used; observe patient for 30 minutes.
    C. RIV or ccIIV should be used.
    D. He should not receive any type of influenza vaccine.
A
  1. Answer: C
    Answers A and B are incorrect because of this patient’s
    history of severe allergy to eggs, as indicated by
    angioedema. If vaccines other than ccIIV4 or RIV are to
    be used, the individual should be vaccinated in a med-
    ical setting supervised by a health care provider who is
    able to recognize and manage severe allergic reactions.
    Answer C is correct because in people with a severe
    egg allergy, ccIIV4 or RIV can be used because they do
    not contain egg protein. Answer D is incorrect because
    the patient is still eligible for an influenza vaccine.